Difference between revisions of "2012 AMC 8 Problems/Problem 18"

m
m (Solution)
Line 5: Line 5:
  
 
==Solution==
 
==Solution==
The problem states that the answer cannot be a perfect square or have prime factors less than <math> 50 </math>. Therefore, the answer will be the product of at least two different primes grater than <math> 50 </math>. The two smallest primes greater than <math> 50 </math> are <math> 53 </math> and <math> 59 </math>. Multiplying these two primes, we obtain the number <math> 3127 </math>, which is also the smallest number on the list of answer choices. So we are done, and the answer is <math> \boxed{\textbf{(A)}\ 3127}</math>.
+
The problem states that the answer cannot be a perfect square or have prime factors less than <math> 50 </math>. Therefore, the answer will be the product of at least two different primes greater than <math> 50 </math>. The two smallest primes greater than <math> 50 </math> are <math> 53 </math> and <math> 59 </math>. Multiplying these two primes, we obtain the number <math> 3127 </math>, which is also the smallest number on the list of answer choices. So we are done, and the answer is <math> \boxed{\textbf{(A)}\ 3127}</math>.
  
 
==See Also==
 
==See Also==
 
{{AMC8 box|year=2012|num-b=17|num-a=19}}
 
{{AMC8 box|year=2012|num-b=17|num-a=19}}
 
{{MAA Notice}}
 
{{MAA Notice}}

Revision as of 14:11, 18 July 2013

Problem

What is the smallest positive integer that is neither prime nor square and that has no prime factor less than 50?

$\textbf{(A)}\hspace{.05in}3127\qquad\textbf{(B)}\hspace{.05in}3133\qquad\textbf{(C)}\hspace{.05in}3137\qquad\textbf{(D)}\hspace{.05in}3139\qquad\textbf{(E)}\hspace{.05in}3149$

Solution

The problem states that the answer cannot be a perfect square or have prime factors less than $50$. Therefore, the answer will be the product of at least two different primes greater than $50$. The two smallest primes greater than $50$ are $53$ and $59$. Multiplying these two primes, we obtain the number $3127$, which is also the smallest number on the list of answer choices. So we are done, and the answer is $\boxed{\textbf{(A)}\ 3127}$.

See Also

2012 AMC 8 (ProblemsAnswer KeyResources)
Preceded by
Problem 17
Followed by
Problem 19
1 2 3 4 5 6 7 8 9 10 11 12 13 14 15 16 17 18 19 20 21 22 23 24 25
All AJHSME/AMC 8 Problems and Solutions

The problems on this page are copyrighted by the Mathematical Association of America's American Mathematics Competitions. AMC logo.png